Which one of the following could be true?

KGregory on August 27, 2020

Please diagram

I have no idea what I am missing here. Please diagram.

Reply
Create a free account to read and take part in forum discussions.

Already have an account? log in

Victoria on August 28, 2020

Hi @KGregory,

Happy to help!

We know that six people each play one of two sports. K, L, M, O, P, and S each play either G or T.

If a sport is played by more than one of the six individuals, then they are ranked in ability from highest to lowest and there are no ties.

G: _ _ _ _ _ _
1 2 3 4 5 6
T: _ _ _ _ _ _
1 2 3 4 5 6

Now let's go through the conditions.

Rule 1 - O plays T.

Rule 2 - L plays G.

Rule 3 - There is no G player ranked higher than L.

Rule 4 - If M plays G, then P and S play G and P ranks lower than M but higher than S.

MG --> PG and SG
Not PG or Not SG --> Not MG

M > P > S

Rule 5 - If M plays T, then S plays T and S ranks lower than O but higher than M.

MT --> ST
Not ST --> Not MT

O > S > M

Rule 6 - If P plays T, then K plays T and O ranks lower than K but higher than P.

PT --> KT
Not KT --> Not PT

K > O > P

So far, we have three different possible scenarios.

Option 1 - M plays G.

G: L > M > P > S
T: O
Unknown: K

Option 2 - M plays T.

G: L
T: O > S > M
Unknown: P & K

Option 3 - P plays T.

G: L
T: K > O > P
Unknown: M & S

Now that we have our diagram and a few possible scenarios outlined, let's go through our answer choices.

We can eliminate answer choice (A) because Rule 5 tells us that if M plays T, then S must play T.

We can eliminate answer choice (B) because Rule 6 tells us that if P plays T, then K must play T.

We can eliminate answer choice (D) because of Rule 6 and Option 3 outlined above. We know that if P plays T, then K plays T and the players must be ranked as follows: K > O > P.

We can eliminate answer choice (E) because of Rule 5 and Option 2 outlined above. We know that if M plays T, then S must play T and the players must be ranked as follows: O > S > M.

Therefore, answer choice (C) should be our correct answer but let's double check. Remember that the correct answer will be a scenario which "could be true" i.e. we just need to outline one possible scenario where it is true.

T: K > O or O > K
G: L > M > P > S

Notice that this meets all of our rules.

1) O plays T.
2) L plays G.
3) There is no G player ranked higher than L.
4) M, P, and S play G with P ranking lower than M but higher than S.
5) This rule is irrelevant because M plays G.
6) This rule is irrelevant because P plays G.

Hope this helps! Please let us know if you have any further questions.